What are the solution(s) to the quadratic equation 50 - x2 = 0?
x = +2,5
x = +63
x = +5,2
no real solution

Answers

Answer 1

Answer:

Step-by-step explanation:

Please use " ^ " for exponentiation:  50 - x^2 = 0.

Solving for x^2, we get:                      x^2 = 50

Taking the square root of both sides:    x = ±√(50), or x = ±7.07

None of the three possible answers here is correct.


Related Questions

Angie bought 44 pencils and paid 66.00. She sold the pencils at a rate of 2 for $3.50
calculate her profit.​

Answers

Answer:

Profit of $11.00

Step-by-step explanation:

44÷2=22

22×$3.50=$77

$77.00-$66.00=$11.00/profit

Given the functions f(x) = 6x + 11 and g(x) = x^2 + 6, which of the following functions represents f[g(x)] correctly?

Answers

Answer:

6x^2+47

Step-by-step explanation:

You simple have to "plug" g(x) into f(x) for x.

f(x)=6x+11

f(g(x))=6(x^2+6)+11

f(g(x))=6x^2+36+11

f(g(x))=6x^2+47

You earn $15 every week .at this rate , how much will you earn in 5 weeks

Answers

Answer:

If you earn $15 every week then you will have earned $75 by the end of 5 weeks.

Step-by-step explanation:

If $15 is 1 week,

and you need to find how much money after 5 weeks,

you multiply by the same number on both sides

1 = 15

1*5 = 15*5

5 = 75

So after 5 weeks you get $75

A 12-centimeter rod is held between a flashlight and a wall as shown. Find the length of the shadow on

the wall of the rod is 45 cm from the wall and 15 cm from the light.

Answers

Answer:

48 cm

Step-by-step explanation:

Given:

Distance of rod from the wall = 45 cm

Distance of rod from the light = 15 cm

Length of rod = 12 cm

We can see that <DAM and <BAF are equal

Also, <DMA and <BFM are equal because they are corresponding angles

To find the length of the shadow, let's take the equation

[tex] \frac{DM}{BF} = \frac{AM}{A.F} [/tex]

Where.:

DM = ½ of length of the rod = ½*12 = 6

A.F = 15 + 45 = 60 cm

AM = 15 cm

Therefore,

[tex] \frac{DM}{BF} = \frac{AM}{A.F} [/tex]

[tex]= \frac{6}{BF} = \frac{15}{60} [/tex]

Cross multiplying, we have:

15 * B.F = 60 * 6

15 * B.F = 360

[tex]BF = \frac{360}{15}[/tex]

BF = 24 cm

The shadow on the wall =

2 * BF

= 2 * 24

= 48 cm

The shadow on the wall is 48 cm

The three hands of an accurate 12-hour clock make a total of _?_ complete revolutions around the clock’s face every 24 hours. (A) 72 (B) 733 (C) 1466 (D) 10 104

Answers

Answer:

(C) 1466

Step-by-step explanation:

The hour hand makes one revolution each 12 hours. So it makes 2 revolutions a day.

The minute hand makes one revolution each hour. So it makes 24 revoulutions a day.

The second hand makes one revolution each minute. So it makes 24*60 = 1440 revolutions a day.

In total:

2 + 24 + 1440 = 1466

The correct answer is C.

y = -x2 + 6x - 5 whats
the axis of symmetry.​

Answers

Answer:

x=3

Step-by-step explanation:

graphed on desmos.com

What is the answer to the series of equations

Answers

Answer:

[tex]71\frac{2}{3}[/tex]

Step-by-step explanation:

1st equation:

3 cups = 3x

3x = 75

x = 25, cup = 25

2d equation

25 + lemon* lemons= 25 + y²

25 + y² = 89

y²= 89 - 25

y²= 64

y = 8, lemon = 8

3d equation

3 circles = 3c

3c = 44

c = 44/3 = 14 2/3

4th equation

25 + 32 + 14 2/3 = 71 2/3

Answer: 427/3

Step-by-step explanation:

The 3 cups added together is equal to 75, therefore, we can establish that each cup is equal to 25.

Cup=25

With the second equation, we plug in 25 for the cup and solve for the circles.

25+ Ο×Ο= 89

Ο×Ο=64

Circle=8

For the third equation, we can find the amount for the ovals if we divide by 3.

Oval= 44/3

Now that we know the value of each picture, we can solve the fourth equation.

25+8×(44/3)=?

25+(352/3)=?

(75/3)+(352/3)=427/3

Which of these leaders would most likely have
spoken these words?
Tecumseh
Thomas Jefferson
Andrew Jackson

Answers

Answer:andrew Jackson

Answer:

D

Step-by-step explanation:

thank the person above ☝☝☝

Edge 2021

Rounded to three decimal places, the value of the irrational number e is?

Answers

Answer:

2.178

hope it helps!

Answer:

2.718

______________________________

One-fourth If a scale factor of 1/4 is used to make a reduction, what is the base of the reduced triangle? ______ cm

Answers

Answer:

  8 cm

Step-by-step explanation:

1/4 of 32 cm is (32/4) cm = 8 cm

The base of the reduced triangle is 8 cm.

Answer:

8 cm

Step-by-step explanation:

I just got it right on edge :) ur welcome

A rectangle is inscribed with its base on the x-axis and its upper corners on the parabola y = 5 − x 2 . What are the dimensions of such a rectangle with the greatest possible area?

Answers

Answer:

A rectangle is inscribed with its base on the x-axis and its upper corners on the parabola

y=5−x^2. What are the dimensions of such a rectangle with the greatest possible area?

Width =

Height =

Width =√10 and Height [tex]= \frac{10}{4}[/tex]

Step-by-step explanation:

Let the coordinates of the vertices of the rectangle which lie on the given parabola y = 5 - x² ........ (1)

are (h,k) and (-h,k).

Hence, the area of the rectangle will be (h + h) × k

Therefore, A = h²k ..... (2).

Now, from equation (1) we can write k = 5 - h² ....... (3)

So, from equation (2), we can write

[tex]A =h^{2} [5-h^{2} ]=5h^{2} -h^{4}[/tex]

For, A to be greatest ,

[tex]\frac{dA}{dh} =0 = 10h-4h^{3}[/tex]

⇒ [tex]h[10-4h^{2} ]=0[/tex]

⇒ [tex]h^{2} =\frac{10}{4} {Since, h≠ 0}[/tex]

⇒ [tex]h = ±\frac{\sqrt{10} }{2}[/tex]

Therefore, from equation (3), k = 5 - h²

⇒ [tex]k=5-\frac{10}{4} =\frac{10}{4}[/tex]

Hence,

Width = 2h =√10 and

Height = [tex]k =\frac{10}{4}.[/tex]

The graph illustrates a normal distribution for the prices paid for a particular model of HD television. The mean price paid is $1600 and the standard deviation is $100. What is the approximate percentage of buyers who paid more than $1900? What is the approximate percentage of buyers who paid less than $1400?What is the approximate percentage of buyers who paid between $1400 and $1600?What is the approximate percentage of buyers who paid between $1500 and $1700?What is the approximate percentage of buyers who paid between $1600 and $1700?What is the approximate percentage of buyers who paid between $1600 and $1900?

Answers

Answer:

(a) 0.14%

(b) 2.28%

(c) 48%

(d) 68%

(e) 34%

(f) 50%

Step-by-step explanation:

Let X be a random variable representing the prices paid for a particular model of HD television.

It is provided that X follows a normal distribution with mean, μ = $1600 and standard deviation, σ = $100.

(a)

Compute the probability of buyers who paid more than $1900 as follows:

[tex]P(X>1900)=P(\frac{X-\mu}{\sigma}>\frac{1900-1600}{100})[/tex]

                   [tex]=P(Z>3)\\=1-P(Z<3)\\=1-0.99865\\=0.00135\\\approx 0.0014[/tex]

*Use a z-table.

Thus, the approximate percentage of buyers who paid more than $1900 is 0.14%.

(b)

Compute the probability of buyers who paid less than $1400 as follows:

[tex]P(X<1400)=P(\frac{X-\mu}{\sigma}<\frac{1400-1600}{100})[/tex]

                   [tex]=P(Z<-2)\\=1-P(Z<2)\\=1-0.97725\\=0.02275\\\approx 0.0228[/tex]

*Use a z-table.

Thus, the approximate percentage of buyers who paid less than $1400 is 2.28%.

(c)

Compute the probability of buyers who paid between $1400 and $1600 as follows:

[tex]P(1400<X<1600)=P(\frac{1400-1600}{100}<\frac{X-\mu}{\sigma}<\frac{1600-1600}{100})[/tex]

                              [tex]=P(-2<Z<0)\\=P(Z<0)-P(Z<-2)\\=0.50-0.0228\\=0.4772\\\approx 0.48[/tex]

*Use a z-table.

Thus, the approximate percentage of buyers who paid between $1400 and $1600 is 48%.

(d)

Compute the probability of buyers who paid between $1500 and $1700 as follows:

[tex]P(1500<X<1700)=P(\frac{1500-1600}{100}<\frac{X-\mu}{\sigma}<\frac{1700-1600}{100})[/tex]

                              [tex]=P(-1<Z<1)\\=P(Z<1)-P(Z<-1)\\=0.84134-0.15866\\=0.68268\\\approx 0.68[/tex]

*Use a z-table.

Thus, the approximate percentage of buyers who paid between $1500 and $1700 is 68%.

(e)

Compute the probability of buyers who paid between $1600 and $1700 as follows:

[tex]P(1600<X<1700)=P(\frac{1600-1600}{100}<\frac{X-\mu}{\sigma}<\frac{1700-1600}{100})[/tex]

                              [tex]=P(0<Z<1)\\=P(Z<1)-P(Z<0)\\=0.84134-0.50\\=0.34134\\\approx 0.34[/tex]

*Use a z-table.

Thus, the approximate percentage of buyers who paid between $1600 and $1700 is 34%.

(f)

Compute the probability of buyers who paid between $1600 and $1900 as follows:

[tex]P(1600<X<1900)=P(\frac{1600-1600}{100}<\frac{X-\mu}{\sigma}<\frac{1900-1600}{100})[/tex]

                              [tex]=P(0<Z<3)\\=P(Z<3)-P(Z<0)\\=0.99865-0.50\\=0.49865\\\approx 0.50[/tex]

*Use a z-table.

Thus, the approximate percentage of buyers who paid between $1600 and $1900 is 50%.

Answer:

Step-by-step explanation:

What is sin(40)?
O A. 0.40
O B. 0.77
O C. 0.64
O D. 0.83

Answers

Answer:

The answer is C.

Step-by-step explanation:

If you type into the calculator for sin(40), you will get approximately 0.64 if you round it to 2 decimal places :

[tex] \sin(40) = 0.6427876097[/tex]

[tex] \sin(40) ≈ 0.64[/tex]

the ratio of bicycles to to cars is 2:7 respectively. If the total number of vehicles are 63, how many bicycles are there?

Answers

Answer:

49 cars and 14 bikes

Step-by-step explanation:

we know [tex]7^{2}[/tex] is 49. 7 × 7 = 49

to match that, 7 × 2 = 14

14 plus 49 is 63

A Poker club has 10 members. A president and a vice-president are to be selected. In how many ways can this be done if everyone is eligible?

Answers

Answer:

90 different ways

Step-by-step explanation:

We have a total of 10 members, and we want to find how many groups of 2 members we can have, where the order of each member in the group of 2 is important, so we have a permutation problem.

To solve this problem, we need to calculate a permutation of 10 choose 2.

The formula for a permutation of n choose p is:

[tex]P(n, p) = n! / (n - p)![/tex]

So we have:

[tex]P(10, 2) = 10! / (10 - 2)![/tex]

[tex]P(10, 2) = 10! / 8![/tex]

[tex]P(10, 2) = 10*9 = 90[/tex]

So there are 90 different ways of choosing a president and a vice-president.

In 2011, the industries with the most complaints to the Better Business Bureau were banks, cable and satellite television companies, collection agencies, cellular phone providers, and new car dealerships (USA Today, April 16, 2012), The results for a sample of 200 complaints are contained in the DATAfile named B8B. Click on the datafile logo to reference the data. a. Construct a frequency distribution for the number of complaints by industry, Category Bank 26 Cable Car Cell 42 60 Collection 28 Total b. Using α = .01, conduct a hypothesis test to determine if the probability of a complaint is the same for the five industries. The test-statistic is p-value- What is your conclusion?c. Which industry has the most complaints?

Answers

Answer:

Step-by-step explanation:

Hello!

The variable of study is

X: number of complaints per industry (Categorized: Bank, Cable, Car, Cell, Collection

Considering this is a categorical variable, and the hypothesis is that all categories have the same probability, you have to apply a Chi-Square  Goodness to Fit test.

Observed frequencies per category

1) Bank: 26

2) Cable: 44

3) Car: 42

4) Cell: 60

5) Collection: 28

Total= 200

Statistical hypotheses:

H₀: P₁=P₂=P₃=P₄=P₅=1/5

H₁: At least one of the proposed proportions is different.

α: 0.01

[tex]X^2= sum \frac{(O_i-E_i)^2}{E_i} ~~X^2_{k-1}[/tex]

For this test the formula for the expected frequencies is:

[tex]E_i= n * P_i[/tex]

So the expected values for each category is:

[tex]E_1= n * P_1= 200*1/5= 40[/tex]

[tex]E_2= n * P_2= 200*1/5= 40[/tex]

[tex]E_3= n * P_3= 200*1/5= 40[/tex]

[tex]E_4= n * P_4= 200*1/5= 40[/tex]

[tex]E_5= n * P_5= 200*1/5= 40[/tex]

[tex]X^2_{H_0}= \frac{(26-40)^2}{40} +\frac{(44-40)^2}{40} +\frac{(42-40)^2}{40} +\frac{(60-40)^2}{40} +\frac{(28-40)^2}{40} = 11.5[/tex]

This test is one tailed and so is its p-value, under a Chi-square with 4 degrees of freedom p-value:  0.021484.

The p-value is less than the significance level so the decision is to reject the null hypothesis.

c. The industry with most complaints is the cellular phone providers

I hope this helps!

What’s the correct answer for this?

Answers

Answer

D.

Explanation

mIH = 44

mIJ = 46

mJK = 77

ADDING THEM ALL TO FIND THE MEASURE OF m<HOK

<HOK = 44+46+77

<HOK = 167°

I think the correct answer is D

The lengths of pregnancies in a small rural village are normally distributed with a mean of 262 days and a standard deviation of 17 days.In what range would you expect to find the middle 68% of most pregnancies

Answers

Answer:

The range in which we can expect to find the middle 68% of most pregnancies is [245 days , 279 days].

Step-by-step explanation:

We are given that the lengths of pregnancies in a small rural village are normally distributed with a mean of 262 days and a standard deviation of 17 days.

Let X = lengths of pregnancies in a small rural village

SO, X ~ Normal([tex]\mu=262,\sigma^{2} = 17^{2}[/tex])

Here, [tex]\mu[/tex] = population mean = 262 days

         [tex]\sigma[/tex] = standard deviation = 17 days

Now, the 68-95-99.7 rule states that;

68% of the data values lies within one standard deviation points.95% of the data values lies within two standard deviation points.99.7% of the data values lies within three standard deviation points.

So, middle 68% of most pregnancies is represented through the range of within one standard deviation points, that is;

[ [tex]\mu -\sigma[/tex] , [tex]\mu + \sigma[/tex] ]  =  [262 - 17 , 262 + 17]

                          =  [245 days , 279 days]

Hence, the range in which we can expect to find the middle 68% of most pregnancies is [245 days , 279 days].

Help people like mE out

Answers

Answer:62.84 cm

Solution,

radius=10 cm

Circumference of circle=2 pi r

=2*3.142*10

=62.84 cm

hope it helps

Good luck on your assignment

Which point on the number line represents the product of 4 and -2.

Answers

Answer:

A: -8

Step-by-step explanation:

4 * -2 is -8, and A is on that point.

Answer:

its a I literally just did it

Step-by-step explanation:

Please answer this correctly

Answers

Answer: The first one

Step-by-step explanation:

If you take a look at the light pink squares, you can count three on the top and three on the bottom. Therefore, when looking at it from the top, you’ll see the first one.

I hope you found my answer helpful! If you did, give it a five-star rating and a thanks! I would really appreciate it!

Even a brainliest if you feel like it! ;)

Answer:

the first image will b da answer

Step-by-step explanation:

happy to help ya :)

One apple and three oranges cost $5.10 and another 5 oranges and one Apple cost $7.50 what is the price of one apple and one orange

Answers

Answer:

$2.7

Step-by-step explanation:

Let the price of one apple be x

and the price of one orange y

Hence ;

One apple and 3 oranges would cost:

x + 3×y = $5.10----------------------------(1)

One apple and 5 oranges would cost;

x + 5×y = $7.50--------------------------(2)

Subtracting eqn (1) from (2), we have :

x-x + 5y -3y = 7.5- 5.1

2y = 2.4

y = $1.2

From eqn(1)

x + 3y = $5.10

x= $5.10-3($1.2) = $5.10-$3.6 = $1.5

Hence x=$1.5 and y= $1.2

We are required to find the cost of one apple and one orange, hence :

x+y = $1.5+$1.2= $2.7

Find the exact coordinates of point E.

Answers

Answer:

is the correct answer.

Step-by-step explanation:

We are given that co-ordinates of D is (-3,8) and F is (5,2).

For finding a point E on the line segment DF dividing it in a ratio 4:1, we can use segment formula.

[tex]x = \dfrac{mx_{2}+nx_{1}}{m+n}\\y = \dfrac{my_{2}+ny_{1}}{m+n}[/tex]

Where [tex](x,y)[/tex] is the co-ordinate of the point which  

divides the line segment joining the points and in the ratio m:n.

As per the given values  

[tex]x_{1} = -3\\x_{2} = 5\\y_{1} = 8\\y_{2} = 2[/tex]

Putting the given values in above formula :

x-co-ordinate of F:

[tex]x = \dfrac{5 \times 4 + (-3 \times 1)}{4+1}\\\Rightarrow \dfrac{17}{5}\\\Rightarrow x = 3.4[/tex]

y-co-ordinate of F:

[tex]y = \dfrac{4 \times 2 + 1 \times 8}{4+1}\\\Rightarrow \dfrac{16}{5}\\\Rightarrow y = 3.2[/tex]

So, answer is [tex]E(3.4,3.2)[/tex].

You bought a sachet water machine from Indie Inc. The cost of the machine was GH¢35,000. At that time, you asked for the payment to be deferred, and a contract was written. Under the contract, you could delay paying for the sachet water machine if you purchased the material for packaging the water from Indie Inc. You will then pay for the machine in a lump sum at the end of 2 years, with interest at a rate of 2% per quarter-year. According to the contract, if you ceased buying the packaging material from Indie Inc. at any time prior to 2 years, the full payment due at the end of 2 years would automatically become due. One year later, you decided to buy the packaging material elsewhere and stopped buying from Indie Inc., whereupon Indie Inc., per the contract terms, asked for the full payment that is due at the end of 2 years to be paid immediately. You were unhappy about this, so Indie Inc. offered as an alternative to accept the GH¢35,000 with interest at 10% per semiannual period for the 12 months that you had been buying the packaging material from Indie Inc. Which of the alternatives should you accept? Explain.

Answers

Answer: The answer has been attached below

Step-by-step explanation:

From the attached calculation, the amount payable in the first payment is cheaper than the amount payable for the second payment.

If both alternatives were presented to me by Indie incorporation after purchasing a sachet water machine from them, I'll choose the first method of payment.

Kindly check the attached file for the calculation which prompted the decision taken.

The cafe where you work just ran out of coffee and you are at the store to buy 1 and 1/2 pounds of coffee you have to put a can with 3/4 pound of coffee in to your shopping cart how many more pounds of coffee do you need

Answers

Answer:

3/4 lbs more

Step-by-step explanation:

1 1/2 - 3/4 = 3/4

You need 3/4 lbs more coffee to have 1 1/2 lbs

The sum of two numbers is 82. One number is 38 more than the other. Find the numbers.

Answers

Answer: 22, 60

Step-by-step explanation:

x+ x + 38 = 82

2x + 38 = 82

2x = 82 - 38

x = 44 / 2 = 22

Answer: 22, 60

Step-by-step explanation:

We can use system of equations to solve this. Let's use 1 for one number and y for the second number.

Equation 1

x+y=82

Equation 2

x+38=y

Since we know what y is, we can substitute it into the first equation.

x+(x+38)=82

x+x+38=82

2x+38=82

2x=44

x=22

Now that we know x, we cna plug it into any equaiton to find y.

(22)+38=y

y=60

What’s the correct answer for this?

Answers

Answer:

20

Step-by-step explanation:

Since chord AB and DE are at equal length from the centre, they are equal in length.

AB = DE

3x-7 = 5(x-5)

3x-7 = 5x-25

-7+25 = 5x -3x

18 = 2x

Dividing both side by 2

x = 9

Now

AB = 3(9)-7

= 27 -7

= 20

A study done by researchers at a university concluded that 70 70​% of all student athletes in this country have been subjected to some form of hazing. The study is based on responses from 1 comma 900 1,900 athletes. What are the margin of error and​ 95% confidence interval for the​ study? The margin of error is nothing .

Answers

Answer:

The margin of error is 0.0206 = 2.06 percentage points.

The 95% confidence interval for the proportion of all student athletes in this country have been subjected to some form of hazing is (0.6794, 0.7206).

Step-by-step explanation:

In a sample with a number n of people surveyed with a probability of a success of [tex]\pi[/tex], and a confidence level of [tex]1-\alpha[/tex], we have the following confidence interval of proportions.

[tex]\pi \pm z\sqrt{\frac{\pi(1-\pi)}{n}}[/tex]

In which

z is the zscore that has a pvalue of [tex]1 - \frac{\alpha}{2}[/tex].

For this problem, we have that:

[tex]n = 1900, \pi = 0.7[/tex]

95% confidence level

So [tex]\alpha = 0.05[/tex], z is the value of Z that has a pvalue of [tex]1 - \frac{0.05}{2} = 0.975[/tex], so [tex]Z = 1.96[/tex].

The margin of error is:

[tex]M = z\sqrt{\frac{\pi(1-\pi)}{n}}[/tex]

[tex]M = 1.96\sqrt{\frac{0.7*0.3}{1900}} = 0.0206[/tex]

The margin of error is 0.0206 = 2.06 percentage points.

The lower limit of this interval is:

[tex]\pi - M = 0.7 - 0.0206 = 0.6794[/tex]

The upper limit of this interval is:

[tex]\pi + M = 0.7 + 0.0206 = 0.7206[/tex]

The 95% confidence interval for the proportion of all student athletes in this country have been subjected to some form of hazing is (0.6794, 0.7206).

help asap !! will get branliest.​

Answers

Answer:

0.8

Step-by-step explanation:

(2 x 10^-4)

= 0.0002

(4x10^3)

= 4000

0.0002 x 4000 = 0.8

Keri deposits $100 in an account every year on the same day. She makes

no other deposits or withdrawals. The account earns an annual rate of 4%

compounded annually. How much interest does it earn on the 2nd year?*

Answers

Answer:

$20.32 or $12.16 depending on the question below!

Step-by-step explanation:

$320.32

It's a bit ambiguous, the current Principal is $100 plus 2 $ additions of $100 or the current Principal is $0 plaus 2 years of $100 contributions. In this case it's $212.16

Other Questions
Under which angle conditions could a triangle exist? Check all that apply.3 acute angles2 acute angles, 1 right angle1 acute angle, 1 right angle, 1 obtuse angle1 acute angle, 2 obtuse angles2 acute angles, 1 obtuse angleO Select the correct answer. Which point lies on the circle represented by the equation (x + 7)2 + (y 10)2 = 132? A. (5,12) B. (-7,-3) C. (-6,-10) D. (6,23) Choose the graph that represents the following set.the whole numbers greater than 3 Solve 9x + 7 = 97 for me please Daley went to the grocery store to buy berries. Blueberries cost $1.50 a pound and strawberries cost $2.25 a pound. Daley spent $10.50 on fruit. The equation he wrote to represent the scenario is 1.5b + 2.25s = 10.5. His solution is: 1. Subtract 2.25s: 1.5b = 10.5 2.25s 2. Divide by 1.5: 1.5b 1.5 = 10.5 2.25s 1.5 3. Distribute 1.5: b = 7 1.5s Daley wrote an equivalent equation to determine how many pounds of blueberries he could buy. Analyze Daleys work. Did he make an error? Yes. In step 1, he had to subtract 1.5b from both sides. Yes. In step 2, he had to multiply 1.5 to both sides. Yes. In step 3, he can only multiply the 1.5 and 12.5 together. No. He correctly solved for b. What was the greatest contribution the Greeks made to Mathematics? Explain. A line parallel to the___ has no defined slope, and a line parallel to the_____ has slope of zero. 1) X-axis. 2) x-axis Y-axis y-axis. Why is an egg unicellular?Because all organisms starts as an eggBecause eggs are the building blocks of life.Because eggs can be seen without a microscopeBecause eggs are made of one cell NEED ASAP WILL MARK BRAINLIEST !! help please i need it!!!!! ill give brainlistWhat is the name of the first ten amendments to the Constitution? Which was the first permanent colony? Who was the president who made the Louisiana purchese??? Please answer this question !! Which best summarizes the pepper moths in England after the industrial revolution Question 1(Multiple Choice Worth 4 points) The star named Canopus has a declination of approximately 52. Which of these statements is correct about Canopus? It is 52 above the celestial equator. It is 52 below the celestial equator. It is 52 to the left of the celestial equator. It is 52 to the right of the celestial equator. 3x +2 = 2x + 13 divided by 3 Please helpppppppp!!!!!Options A- enable movement, provide energy, store energy B- enable movement, provide energy, store energy C- enable movement, provide energy, store energy D- DNA, cholesterol, starches E- DNA, cholesterol, starches Solve the equation below for x.4(2x +8) - 3x = 471.X= 132.X= 23.X= 34.X= 7.8 The rectangle has a length of 4x+3 and a width of 3x. Show your work.A. Find the perimeter. B. Find the area. C. Find the perimeter and area if x = 8. 10 moles of carbon dioxide has a mass of 440 g. What is the relative formula mass of carbon dioxide? What is the inverse of the function f(x)= 6x-5/x+9 The MOST LIKELY impact of the Columbian Exchange was the transfer of ______. NEED HELP!